K
Khách

Hãy nhập câu hỏi của bạn vào đây, nếu là tài khoản VIP, bạn sẽ được ưu tiên trả lời.

16 tháng 7 2016

a) Theo đầu bài ta có:
\(\orbr{\begin{cases}\frac{n}{n+1}=\frac{n\left(n+4\right)}{\left(n+1\right)\left(n+4\right)}=\frac{n^2+2n+2n}{\left(n+1\right)\left(n+4\right)}\\\frac{n+1}{n+4}=\frac{\left(n+1\right)\left(n+1\right)}{\left(n+1\right)\left(n+4\right)}=\frac{n^2+2n+1}{\left(n+1\right)\left(n+4\right)}\end{cases}}\)
Nếu \(n=0\Rightarrow2n=0< 1\Rightarrow\frac{n^2+2n+2n}{\left(n+1\right)\left(n+4\right)}< \frac{n^2+2n+1}{\left(n+1\right)\left(n+4\right)}\Rightarrow\frac{n}{n+1}< \frac{n+1}{n+4}\)
Nếu \(n\ge1\Rightarrow2n\ge2>1\Rightarrow\frac{n^2+2n+2n}{\left(n+1\right)\left(n+4\right)}>\frac{n^2+2n+1}{\left(n+1\right)\left(n+4\right)}\Rightarrow\frac{n}{n+1}>\frac{n+1}{n+4}\)

16 tháng 7 2016

Bài 2

 \(a,\left(x-3\right)^2=9\Leftrightarrow\left(x-3\right)^2=3^2\Leftrightarrow x-3=3\Leftrightarrow x=6\)

\(b,\left(\frac{1}{2}+x\right)^2=16\Leftrightarrow\left(\frac{1}{2}+x\right)^2=4^2\Leftrightarrow\frac{1}{2}+x=4\Leftrightarrow x=\frac{7}{2}\)

Số thập phân vô hạn tuần hoàn tạp: 

+) Lấy số tạo bởi phần bất thường và chu kì trừ đi phần bất thường làm tử. 
+) Mẫu số là số gồm các chữ số 9 và kèm theo là các chữ số 0; số chữ số 9 bằng số chữ số trong chu kỳ, số chữ số 0 bằng số chữ số của phần bất thường. 

Vậy \(0,1\left(02\right)=\frac{102-1}{990}=\frac{101}{990}\)nha bạn!

16 tháng 7 2016

????????????????????????????

16 tháng 7 2016

Gọi 3 phân số tối giản cần tìm là a/b, c/d và e/f. Theo đầu bài ta có:
\(\frac{a}{b}+\frac{c}{d}+\frac{e}{f}=5\frac{25}{63}=\frac{340}{63}\) ( 1 )
Do a, c, e tỉ lệ nghịch với 20 ; 4 ; 5 nên \(a:c:e=1:5:4\Rightarrow a=\frac{c}{5}=\frac{e}{4}\Rightarrow\hept{\begin{cases}c=5a\\e=4a\end{cases}}\) ( 2 )
Do b, d, f tỉ lệ thuận với 1 ; 3 ; 7 nên \(b:d:f=1:3:7\Rightarrow b=\frac{d}{3}=\frac{f}{7}\Rightarrow\hept{\begin{cases}d=3b\\f=7b\end{cases}}\) ( 3 )
Thế ( 2 ), ( 3 ) vào 1, ta có:
\(\frac{a}{b}+\frac{5a}{3b}+\frac{4a}{7b}=\frac{340}{63}\)
\(\Rightarrow1\cdot\frac{a}{b}+\frac{5}{3}\cdot\frac{a}{b}+\frac{4}{7}\cdot\frac{a}{b}=\frac{340}{63}\)
\(\Rightarrow\frac{a}{b}\cdot\left(1+\frac{5}{3}+\frac{4}{7}\right)=\frac{340}{63}\)
\(\Rightarrow\frac{a}{b}\cdot\frac{68}{21}=\frac{340}{63}\)
\(\Rightarrow\hept{\begin{cases}\frac{a}{b}=\frac{340}{63}:\frac{68}{21}=\frac{5}{3}\\\frac{c}{d}=\frac{5a}{3b}=\frac{25}{9}\\\frac{e}{f}=\frac{4a}{7b}=\frac{20}{21}\end{cases}}\)

30 tháng 11 2018

e/f đã tính đâu

16 tháng 7 2016

\(\frac{\left(0,6\right)^5.\left(0,3\right)^3}{\left(0,2\right)^6.\left(0,3\right)^7}\)

\(=\frac{\left(\frac{6}{10}\right)^5.\left(\frac{3}{10}\right)^3}{\left(\frac{2}{10}\right)^6.\left(\frac{3}{10}\right)^7}\)

\(=\frac{6^5.3^3.\frac{1}{10^8}}{2^6.3^7.\frac{1}{10^{13}}}\)

\(=\frac{2^5.3^5.3^3}{2^6.3^7.\frac{1}{10^5}}\)

\(=\frac{10^5.3}{2}\)

\(=150000\)

16 tháng 7 2016

Ta thấy các phân số ở tổng A khi quy đồng mẫu số sẽ chứa lũy thừa của 2 với số mũ lớn nhất là 2k (2k < hoặc = n) như vậy khi quy đồng mẫu số thì các phân số đều có tử chẵn chỉ có phân số 1/2k có tử lẻ

=> A có tử lẻ mẫu chẵn, không là số nguyên

=> đpcm

16 tháng 7 2016

A không phải là số nguyên vì:

   + Số 1 là 1 số nguyên, (không được là số thập phân)

   + Số 1 được cộng vời các số còn là phân số 

Ta cũng thấy rằng bất cứ một số nguyên  nào mà cộng vời lại phân số thì kết quả chắc chắn là 1 phân số, bạn cứ thử đi sẽ thấy.

  Từ những điều trên ta => A không pải là số nguyên.

16 tháng 7 2016

\(\frac{x-1}{2}=\frac{y+3}{4}=\frac{z-5}{6}\)và 5z-3x-4y=50

ta có:

\(\frac{x-1}{2}=\frac{y+3}{4}=\frac{z-5}{6}=>\frac{3x-3}{6}=\frac{4y+12}{16}=\frac{5z-25}{30}\)

\(=\frac{5z-25-\left(3x-3\right)-\left(4y+12\right)}{30-16-6}\)\(=\frac{5z-25-3x+3-4y-12}{8}=\frac{5z-3x-4y-34}{8}\)

\(=\frac{50-34}{8}=\frac{16}{8}=2\)

Vậy x=5;y=5;z=17

16 tháng 7 2016

Theo đầu bài ta có:
\(\frac{x-1}{2}=\frac{y+3}{4}=\frac{z-5}{6}\)
\(\Rightarrow\frac{3\left(x-1\right)}{3\cdot2}=\frac{4\left(y+3\right)}{4\cdot4}=\frac{5\left(z-5\right)}{5\cdot6}\)
\(\Rightarrow\frac{3x-3}{6}=\frac{4y+12}{16}=\frac{5z-25}{30}\)
\(=\frac{\left(5z-25\right)-\left(3x-3\right)-\left(4y+12\right)}{30-6-16}\)
\(=\frac{\left(5z-3x-4y\right)-\left(25-3+12\right)}{8}\)
\(=\frac{50-34}{8}=\frac{16}{8}=2\)
\(\Rightarrow\hept{\begin{cases}x=2\cdot2+1=5\\y=2\cdot4-3=5\\z=2\cdot6+5=17\end{cases}}\)